LSAT and Law School Admissions Forum

Get expert LSAT preparation and law school admissions advice from PowerScore Test Preparation.

 reop6780
  • Posts: 265
  • Joined: Jul 27, 2013
|
#10135
On the page of 20, page number 2, I cannot distinguish answer choices among A, C, and D. I was trying to approach this by conditional reasoning: military deterrence -> retaliatory power. However, it does not make any difference among A, C, and D.

A: deterred -> retaliation (only if)
C: fails to attack (military deterrence) -> retaliatory attack
D: deterrence -> retaliatory attack

They all fit in to the original condition. Also, the correct answer is stated as D, and I was between A and C in that D includes texts such as "the interests of," and "unsurpassed military power." I regarded such information as out of context. Did I approach this question in wrong direction or missed some information to be rephrased?

Thank you
 Steve Stein
PowerScore Staff
  • PowerScore Staff
  • Posts: 1153
  • Joined: Apr 11, 2011
|
#10137
Hi reop6780,

That's a great question. I would suggest spelling out the necessary condition a bit further:

To maintain military deterrence, according to the theory and the passage, "a nation would have to be believed to have retaliatory power so great that a potential aggressor would have reason to think it could not defend itself..."

Deter :arrow: thought to have retaliatory power that could not be defended against

Answer choice A claims that deterrence can be achieved only with certain knowledge of retaliatory destruction in response to an attack. This is very different from the necessary condition presented in the stimulus, that the potential aggressor merely must have reason to think that it could not defend itself against retaliation.

As for answer choice C, a nation that has not attacked another has not necesssarily been deterred; the lack of an attack is not the same as deterrence (you have not attacked me, for example, but that does not necessarily mean that I have been actively deterring such an attack).

Answer choice D is the right answer in this case: If a nation has unsurpassed military power and seeks deterrence, it is a good idea to give would-be aggressor nations reason to think that an attack would result in retaliation that could not be defended against.

(Deter :arrow: thought to have retaliatory power that could not be defended against)

I hope that's helpful! Let me know--thanks!

~Steve
 reop6780
  • Posts: 265
  • Joined: Jul 27, 2013
|
#10140
Hi Steve !

It helped me a lot!

Thanks !

Hyun Kim
 haideemaria
  • Posts: 1
  • Joined: Apr 04, 2014
|
#14488
Hello,

Currently I am working on Must Be True Questions and I wanted a few explanations for wrong answer choices, as the book does not provide any. All of these questions are from the Logical Reasoning Question Type Training book.

pg 20, Q2
The correct answer is D.
I choose answer C.
 Robert Carroll
PowerScore Staff
  • PowerScore Staff
  • Posts: 1787
  • Joined: Dec 06, 2013
|
#14498
haidee,

As with all Must Be True questions, an answer choice that includes new information, or something that merely COULD be true, is not correct - the right answer has to follow from the information in the stimulus.

Answer A speaks about "certain knowledge" where the stimulus only requires a nation to be "believed" to have great retaliatory power. This answer choice adds new information because the stimulus did not require knowledge, only belief.

Answer B is wrong because the theory of military deterrence discussed in the stimulus is about the retaliatory power of the country a potential aggressor is considering attacking; it is not about the potential aggressor's retaliatory power. So this is new information again.

Answer C is wrong because the theory of military deterrence is one possible reason a potential aggressor would not attack another country. The theory does not exclude other possible reasons for a country not to attack another. C incorrectly claims that all failures to attack are examples of deterrence.

Answer E is wrong because it includes new information, claiming that a nation needs greater retaliatory force than any other nation when this is not anywhere in the stimulus itself.

Robert
 ovibalaj
  • Posts: 8
  • Joined: Jun 22, 2015
|
#18980
"Logical Reasoning - Question Type Training" Chapter 2: Must Be True; Question 2; page 20.

Stimulus:

"The theory of military deterrence was based on a simple psychological truth....."

Question Stem:

"If the statements above are true, which one of the following can be properly inferred"?

** Inferred in LSAT = must be true


Analysis [Primary Objectives]:

* P.O 1. Determine whether the stimulus contains an argument or if it is only a set of factual statements.

"The theory of military deterrence was based on a simple psychological truth...."


Clearly – conclusion indicator


This stimulus is an Argument.

* P.O 2. If it is an argument, identify the conclusion of the argument. If it is a fact set, examine each fact.

This stimulus is an Argument. It's main conclusion is:

This is preceded by the conclusion indicator – CLEARLY

" Clearly, then, to maintain military deterrence, a nation would have to be believed to have retaliatory power so great that a potential aggressor nation would have reasons to think that it could not defend itself against such retaliation."


* P.O 3. If the stimulus contain an argument, determine if the argument is strong or weak.

The argument is strong. What would be the reason why this is?

* P.O 4. Read closely and know precisely what the author said. Do Not Generalize.

It seems that it is summed up in the conclusion. Isn't the conclusion always going to fulfill this Primary Objective?

* P.O 5. Carefully read and identify the question stem. Do not assume that certain words are automatically associated with certain types.

"If the statements above are true, which one of the following can be properly inferred"?

It is asking which one of the answers must be true.


* P.O 6. After reading the question stem, take a moment to mentally formulate your answer to the question stem. [PREPHRASE]

What should I do here?


* P.O 7. Always read each five of the answer choices.

Done…

* P.O 8. Separate the answer choices into Contenders and Losers. After you complete the process, review the contenders and decide which answer is the correct one.

Stimulus:

"The theory of military deterrence was based on a simple psychological truth..."

Question Stem:

"If the statements above are true, which one of the following can be properly inferred"?



Answer Choices:

A. A would-be aggressor nation can be deterred from attacking only if it has certain knowledge that it would be destroyed in retaliation by the country it attacks.

LOSER
→ It's language is too strong. The phrase "destroyed in retaliation" goes beyond the statement of the stimulus. It is retaliation alone that can cause the aggressor to hesitate an attack.

B. A nation will not attack another nation if it believes that its own retaliatory power surpasses that of the other nation.

LOSER
→ this seem to go against the initial premise of the stimulus. It is completely backwards of the theory.

C. One nation's failing to attack another establishes that the nation that fails to attack believes that it could not withstand a retaliatory attack from other nation.

LOSER
→ Again it seems that from the stimulus, fear of retaliation alone causes the aggressor to withhold an attack, not fear of being destroyed in retaliation. Although this would probably be a logical assumption to make, in a MUST BE TRUE question, you must use the authors statements as fact, not your own conceptions about reality.


D. It is in the interests of a nation that seeks deterrence and has unsurpassed military power to let potential aggressors against it become aware of its power of retaliatory attack.

CONTENDER
→ This is sort of a summary of the conclusion of the authors argument. It encapsulates the theory and stays within the set parameters in the stimulus.

E. Maintaining maximum deterrence from aggression by other nations require that a nation maintain a retaliatory force greater than that of any other nation.

CONTENDER?
→ I do not know why this is wrong per say. Could you help me on this?
 Nikki Siclunov
PowerScore Staff
  • PowerScore Staff
  • Posts: 1362
  • Joined: Aug 02, 2011
|
#18983
Hi ovibalaj,

Once again, please do NOT post copyrighted material on the forum—we had to remove it! Regardless, please see my comments below. Thanks! :)
* P.O 3. If the stimulus contain an argument, determine if the argument is strong or weak.

The argument is strong. What would be the reason why this is?
To evaluate argument strength, as yourself if you are convinced by the author's line of reasoning. How well do the premises support the conclusion? Do you see any gaps in the reasoning, or unwarranted assumptions? This step is admittedly more relevant to questions in the Help and Hurt families, where you are expected to either strengthen or weaken the argument in the stimulus. Argument evaluation is less important when it comes to MBT questions.
* P.O 4. Read closely and know precisely what the author said. Do Not Generalize.

It seems that it is summed up in the conclusion. Isn't the conclusion always going to fulfill this Primary Objective?
You might have misunderstood the objective: the goal isn't to summarize the argument, but rather to understand precisely its scope and limits. Basically, it's a cautionary reminder to read carefully. It's not step-by-step "objective" in the strict sense of the word. If you're capable of reading closely, as you seem to be, you shouldn't worry about this step too much.
* P.O 6. After reading the question stem, take a moment to mentally formulate your answer to the question stem. [PREPHRASE]

What should I do here?
When it comes to MBT questions, we typically prephrase the answer to those questions that are based on fact sets: the prephrase is usually the conclusion that can be inferred on the basis of these facts. As you pointed out, here we already have a conclusion; consequently, no prephrase is necessary. Just make sure to understand what the conclusion is, and its implications: the policy of military deterrence relies on each party's belief (valid or not) that the other party is capable of nuking them. That's all.

Your analysis of answer choices (A) through (D) is correct. Regarding (E), maintaining maximum deterrence does not necessarily require a retaliatory power greater than that of any other nation: it simply requires other nations to believe that you possess such power. Does that make sense?

Thanks!
 ovibalaj
  • Posts: 8
  • Joined: Jun 22, 2015
|
#18984
Nikki Siclunov wrote:Your analysis of answer choices (A) through (D) is correct. Regarding (E), maintaining maximum deterrence does not necessarily require a retaliatory power greater than that of any other nation: it simply requires other nations to believe that you possess such power. Does that make sense?
I do understand. Thank you for the help.

Sincerely, Ovi
 sblum
  • Posts: 4
  • Joined: Jun 12, 2017
|
#35958
Can someone explain to me how this prompt is a case of mistaken reversal? :-?
 Lawyered
  • Posts: 23
  • Joined: Jun 13, 2017
|
#36019
sblum wrote:Can someone explain to me how this prompt is a case of mistaken reversal? :-?

Yeah, I would appreciate this as well.

And what is the "flaw" in the argument? The top comment mentions the flaw but I fail to see it.

Thanks!

Get the most out of your LSAT Prep Plus subscription.

Analyze and track your performance with our Testing and Analytics Package.